Please help!!!
Find measure QR.

Please Help!!!Find Measure QR.

Answers

Answer 1

Answer:

  QR = 20

Step-by-step explanation:

You want the measure of side QR of isosceles triangle PQR with congruent sides PQ and QR marked 5x and 3x+8, respectively.

Congruent sides

The sides of the isosceles triangle are congruent, so we have ...

  PQ = QR

  5x = 3x +8

  2x = 8 . . . . . . . subtract 3x

  x = 4 . . . . . . . divide by 2

  QR = 3x +8 = 3(4) +8 = 20 . . . . . . find the length of QR

The measure of QR is 20 units.

__

Additional comment

You will recognize that the segment QS is both the altitude and median of the triangle. That means the triangle is isosceles, so symmetrical about QS.

If you need a formal proof, you can observe that QS is congruent to itself, so the two right triangles are congruent by LL (or SAS).


Related Questions

What is x+3y ≥ 3
Help?

Answers

Answer: x

3

3

y

Step-by-step explanation:

Select the correct answer from each drop-down menu. which formulas return true, if the value in cell b4 is 12 and the value in c4 is 29? and return true, if the value in cell b4 is 12 and the value in c4 is 29.

Answers

the correct answer from each drop-down menu. which formulas return true,.if the value in cell b4 is 12 and the value in c4 is 29.: A. =B4=12 AND C4=29

The correct answer is A. =B4=12 AND C4=29, as this is the only formula that returns true if the value in cell b4 is 12 and the value in c4 is 29.

Both B. =(B4=12) AND (C4=29) and D. =(B4+C4)=41 are correct, but they do not fit the given criteria.

C. =B4+C4=41 does not return true for the given values, as 12 + 29 does not equal 41.

Learn more about formula here

https://brainly.com/question/1293486

#SPJ4

A system of equations consists of a line s of the equation y = x – 5 and a line t that passes through the points (0, 2) and (8, –4). Answer the questions about line t to write the equation.

What is the slope of line t?

–0.75
What is the y-intercept of line t?

2
What is the equation in slope-intercept form of line t?

y = –0.75x + 2

Answers

The line 't' is y = -0.75x + 2 which has a slope of negative 0.75 and the y-intercept is 2.

What is a linear equation?

A connection between a number of variables results in a linear model when a graph is displayed. The variable will have a degree of one.

The linear equation is given as,

y = mx + c

Where m is the slope of the line and c is the y-intercept of the line.

The line t that passes through the points (0, 2) and (8, -4) is given as,

(y - 2) = [(- 6 - 2) / (8 - 0)](x - 0)

y - 2 = - 0.75x + 0

y = -0.75x + 2

The line 't' is y = -0.75x + 2 which has a slope of negative 0.75 and the y-intercept is 2.

More about the linear equation link is given below.

https://brainly.com/question/11897796

#SPJ1

[tex]\sqrt{x} \sqrt{x} 100\\[/tex]

Answers

Answer:

100 isn't under the root, so the roots over the x cancel and you have 100x as the answer

Step-by-step explanation:

100 isn't under the root, so the roots over the x cancel and you have 100x as the answer

Which graph represents function f?
f(x) = 2|x − 1|– 3

Answers

Answer:

graph D

Step-by-step explanation:

if x = 0

y = 2 |-1| - 3

y = 2 (1) - 3

y = 2 - 3

y = -1


A (0, -1)


if x = 1

y = 2 |1-1|  - 3

y = 2 (0) - 3

y = -3


B (1,-3)

find the number a such that the limit exists. lim x→−2 4x2 + ax + a + 12 x2 + x − 2

Answers

The limit exists and equals -4 when a = 28.

How to illustrate the limit?

From the information, we want to find the number a such that the limit exists. lim x→−2 4x² + ax + a + 12 divided by x² + x − 2.

It should be noted that this will be illustrated as:

(4x² + ax + a + 12) / (x² + x − 2)

We will substitute x = -2 into the value of x in the Illustration. This will be:

(16 - 2a + a + 12) / (4 - 2 - 2)

16 - 2a + a + 12 = 0

-a = -28

a = 28

Using lhopital rule, we'll find the derivative. This will be:

8x + a / 2x + 2

Substitute x = -2

(-16 + 28) / (-4 + 1)

= 12 / -3

= -4

Therefore the limit exists and equals -4 when a = 28.

Learn more about limit on

brainly.com/question/282767

#SPJ1

Complete question

find the number a such that the limit exists. lim x→−2 4x² + ax + a + 12 divided by x² + x − 2

What is the solution to this system of linear equations?
2x + y = 1
3x - y = -6

Answers

Answer:

x = -1

y = 3

Step-by-step explanation:

2x + y = 1

3x - y = -6

Let's find x first by adding both equations

5x = -5

x = -1

Now put -1 in the spot of the variable x

2(-1) + y = 1

-2 + y = 1

y = 3

a line with a slope of 2 passes through the points (10,8) and (w,6). what is the value of w?

Answers

Answer:

w = 9

Step-by-step explanation:

Use a slope formula to solve. Subtract the y's put that on top and subtract x's and put that on the bottom. This calc should equal 2 that was given.

(8-6)/(10-w) = 2

2/(10-w) = 2

Cross-multiply.

2 = 2(10-w)

Distributive property

2 = 20 - 2w

Add 2w

2 + 2w = 20

Subtract 2

2w = 18

divide by 2

w = 18/2 = 9

Or you can logic it out...8-6 is 2 on top. Need a 1 on the bottom (bc 2/1 is 2) 10-9 is 1 for on the bottom. w = 1

identify the value of f(x) and x from the given coordinates

Answers

Answer:

1

Step-by-step explanation:

For function f(x), the average rate of change from x = a to x = b is:

average rate of change = (f(b) - f(a))/(b - a)

Here we have:

f(a) = f(-3) = -2.88

f(b) = f(2.5) = 2.62

average rate of change = (2.62 - (-2.88))/(2.5 - (-3)) = 1

Answer the questions about the following polynomial..... please help!

Answers

Answer:

x⁵ - 1/3x² -1

Step-by-step explanation:

In order to write polynomials in standard form:

1) Find the degree (power) of each term

2) Arrange them in descending order (largest to smallest)

We have : -1 + x⁵ - 1/3x²

The degree of -1 is 0 since there is no power.

The degree of x⁵ is 5.

The degree of - 1/3x² is 2.

Now arranging them in descending order:

x⁵ has the largest degree, followed by - 1/3x² and then -1, so the polynomial in standard form is:

x⁵ - 1/3x² -1

The prices paid for a model of a new car are approximately normally distributed with a mean of $17,000 and a standard deviation of $500.

The price that is 3 standard deviations above the mean is $_

The price that is 2 standard deviations below the mean is $_

The percentage of buyers who paid between $16,500 and $17,500 is %___

The percentage of buyers who paid between $17,000 and $18,000 is %___

The percentage of buyers who paid less than $16,000 is %__

Answers

Answer: If the prices paid for a model of a new car are approximately normally distributed with a mean of $17,000 and a standard deviation of $500, it means that the majority of the prices paid for the car will fall within a certain range around the mean. Specifically, approximately 68% of the prices paid will be within one standard deviation of the mean, which in this case would be between $16,500 and $17,500. Approximately 95% of the prices paid will be within two standard deviations of the mean, which would be between $16,000 and $18,000. And approximately 99.7% of the prices paid will be within three standard deviations of the mean, which would be between $15,500 and $18,500. This shows that the prices paid for the car are relatively consistent, with only a small percentage falling outside of the range determined by the mean and standard deviation.

Step-by-step explanation:

a fast-food restaurant runs a promotion in which certain food items come with game pieces. according to the restaurant, 1 in 4 game pieces is a winner. if jeff keeps playing until he wins a prize, what is the probability that he has to play the game exactly 5 times?

Answers

The probability that Jeff has to play the game exactly 5 times i.e he keeps playing until he wins prize is (0.25)⁵.

We have, a fast-food restaurant runs a promotion which consists some food items with games pieces. 1 in 4 game pieces is a winner. This means, probability of winning game or sucess (p) = 1/4 . Since number of trials is fixed, trials are independent and probability of success is constant in each trial, we can use Binomial distribution. The Binomial distribution probability formula is

P(X= x) = ⁿCₓ(p)ˣ(1-p)⁽ⁿ⁻ˣ⁾

where,n --> number of trials

p --> probability of success

x --> number of times for a specific outcome within n trials

ⁿCₓ --> number of combinations

we have calculate the probability that he has to play the game exactly 5 times, i.e

x = 5 . Jeff keeps playing until he wins and he wins when he play exactly 5 times so,n= 5

Now, plugging all known values in above formula we get,

P(X= 5) = ⁵C₅(0.25)⁵(1-0.25)⁰

=> P(X= 5) = ⁵C₅(0.25)⁵(0.75)⁰

=> P(X= 5) = 1× (0.25)⁵× 1 = (0.25)⁵

Hence, required probability is (0.25)⁵

To learn more about binomial Probability distribution, refer:

https://brainly.com/question/9325204

#SPJ4

For a recipe, harris uses 2 cups of sugar for each cup of flour. how many cups of flour does harris need when he uses 1 cup of sugar?

Answers

Answer:

1/2 a cup of flour

Step-by-step explanation:

Sugar to flour ratio:
2 to 1
That means for every 2 sugar Harris needs 1 cup flour
There are twice as many cups of sugar of cups of flour
Since 1/2 times 2 is 1 that means the ratio is ture

2 to 1 = 0.5 to 1

Hope this helps :) im kinda bad at explaining

What is the 5th term in this sequence? 1, -8, -17

Answers

The value of the fifth term in the given sequence as represented in the task content is; -35.

What is the value of the fifth term in the sequence as given?

It follows from the task content that the value of the fifth term in the given sequence is to be determined.

On this note, it follows from the observation that the given sequence is arithmetic and the nth term, T (n) = a + ( n - 1 ) d.

where, a = first term = 1

and d = common difference = -8 - 1 = -17 - (-8) = -9.

Therefore, the fifth term is given as;

T (5) = 1 + ( 5 - 1 ) -9

= 1 + (4 × -9)

= 1 - 36

= -35.

On this note, the fifth term of the sequence as required is; -35.

Read more on arithmetic sequence;

https://brainly.com/question/6561461

#SPJ1

a company wants to encrypt a document containing important passwords. to do this, the sum of two positive numbers will need to be minimized. if the product of both numbers is 47, what is the minimum sum?

Answers

The requried, "a" and "b" are positive numbers, the minimum sum (a + b) is 13.711.

To find the minimum sum of two positive numbers whose product is 47, we can use the concept of the arithmetic mean-geometric mean inequality (AM-GM inequality).

The AM-GM inequality states that for any two positive numbers, the arithmetic mean (average) is always greater than or equal to the geometric mean. Mathematically, it can be expressed as:

AM ≥ GM

For two positive numbers "a" and "b," the arithmetic mean is (a + b) / 2, and the geometric mean is √(ab).

Given that the product of the two numbers is 47 (ab = 47), we want to find the minimum value of their sum (a + b).

Using the AM-GM inequality:

(a + b) / 2 ≥ √(ab)

Substitute ab = 47:

(a + b) / 2 ≥ √47

Now, let's solve for the minimum sum (a + b):

a + b ≥ 2(√47)

a + b ≥ 2 * √(47)
a + b ≥  13.711.        

Since "a" and "b" are positive numbers, the minimum sum (a + b) is 13.711.

Learn more about the minimum sum of numbers here:

https://brainly.com/question/33367738

#SPJ12

8.
Are the two ratlos below proportional? *
5
15
6' 18
Mark only one oval.
a. No, because the unit rates are not equal.
b. Yes, because the unit rates are the same.
c. No, because the rates are the same.
d. Yes, because the unit rates are not equal.
2 points

Answers

Made possible via methods. In light of this, 5, 6, 15, and 18 are in proportion. The same unit rates prove that.

Rates and proportions are they equivalent?

A ratio, on the other hand, is the comparison of the sizes of two values, whereas a proportion is a part, share, or quantity in relation to an overall total. Each of the three measures has unique characteristics, and a rate is just one number divided by another.

The same units apply to proportions, or not?

Due to the consistency of the units and their equivalent, each ratio compares the same units—inches and feet—and is therefore similar. Compare two ratios using the same units when using proportions.

To know more about proportions visit:

https://brainly.com/question/29765554

#SPJ1

select the correct answer from each drop down menu

choices: the rate of function eight is (greater than), (less than), (equal to) the rate of change a function b

(neither function is ), (only function b is) , (only function a is) , (both functions are) increasing.

The y intercept of function a is (less than) , (greater than), (equal to) the y-intercept of function B

whatever is in the parentheses are the choices. please help

Answers

The rate of change of function A is greater than the rate of change of function B.

The y-intercept of function A is less than the y-intercept of function B.

How to calculate the rate of change of the function?

The rate of change is also called the slope of a linear equation and is defined as the rate of change of y-values divided by the corresponding change in x-values.

For function A, we are told that the line passes through the points (2, 3) and (-1, -3). The rate of change is gotten from the formula;

Rate of change = (y₂ - y₁)/(x₂ - x₁)

Rate of change = (-3 - 3)/(-1 - 2)

Rate of change = -6/-3

Rate of change = 2

For function B, we are given that the equation of the line is;

y = ¹/₂x + 2

Now, formula for equation of a line in slope intercept form is;

y = mx + c

where ;

m is slope and c is y-intercept

Thus, slope is 1/2 and y-intercept is 2

For function A, we can find the y-intercept as;

3 = 2(2) + c

c = 3 - 4

c = -1

Read more about Rate of change at; https://brainly.com/question/8728504

#SPJ1

please solve theta
csc^2 theta=cot theta+3

Answers

The value of θ will -45° or 26.56°

What are trigonometric identities?

Trigonometric Identities are the equalities that involve trigonometry functions and holds true for all the values of variables given in the equation.

Given that, cosec²θ = cotθ + 3

Solving for θ,

∵ cosec²θ = cot²θ+1

∴ cot²θ+1 = cotθ + 3

cot²θ-cotθ-2 = 0

Factorizing and solving, we get

(cotθ+1)(cotθ-2) = 0

cotθ+1 = 0

cotθ = -1

θ = -45°

or, cotθ-2 = 0

cotθ = 2

θ = 26.56°

Hence, The value of θ will -45° or 26.56°

For more references on Trigonometric Identities, click;

https://brainly.com/question/24377281

#SPJ1

HELPPPP!! how do i find the angle measure ?

Answers

Answer:

1. m<AOB = 50 degree

2. m<COD = 90 degree

3. m<BOD = 130 degree

4. m<AOD = 180 degree

Step-by-step explanation:

1. We see that <AOB has one leg at 0 and the other leg at 50 degrees, so the <AOB is 50 degrees.

2. We see that <COD has one leg at 0 and the other leg at 90 degrees, so the <COD is 90 degrees

3. We see that <BOD has one leg at 50 and the other leg at 180 leg, so the <BOD is 180 - 50 = 130 degree

4/ We see that <AOD has one leg at 0 and the other leg at 180 degrees, so the <AOD is 180 - 0 = 180 degrees

There are 30 giraffe and 6 penguin at the zoo. Which tatement correctly compare the two quantitie?

Answers

To compare the two quantities, divide the number of giraffes by the number of penguins There are 5 times as many giraffes as penguins at the zoo.

To compare the two quantities, you need to divide the number of giraffes by the number of penguins.

30 giraffes / 6 penguins = 5

This means that there are 5 times as many giraffes as penguins at the zoo.

There are 5 times as many giraffes as penguins at the zoo. To compare the two quantities, divide the number of giraffes by the number of penguins (30/6 = 5). This means that there are 5 times more giraffes than penguins at the zoo.

Learn more about number here

https://brainly.com/question/10547079

#SPJ4

if x^2 + y^2 = 289, find the value of dy/dt at (8,15)

Answers

Answer: a)

Step-by-step explanation:

To find [tex]\frac{dy}{dx}[/tex], we will have to use implicit differentiation, and because the base is [tex]dx[/tex], we will take the deriviative of both sides with respect to [tex]x[/tex].

To derive [tex]y[/tex] with respect to [tex]x[/tex], just derive the term with respect to y then multiply the term with [tex]\frac{dy}{dx}[/tex].

So [tex]\frac{d}{dx} x^{2}[/tex] is just [tex]2x[/tex], [tex]\frac{d}{dx} y^{2}[/tex] is [tex]2y \frac{dy}{dx}[/tex], and  [tex]\frac{d}{dx} 289[/tex] is 0. Now substitue each term with it's deriviative.

[tex]2x + 2y \frac{dy}{dx} = 0[/tex]

Now, just solve for  [tex]\frac{dy}{dx}[/tex] when [tex]x[/tex] is 8 and [tex]y[/tex] is 15

[tex]2(8) + 2(15)\frac{dy}{dx} = 0[/tex]

[tex]16 + 30\frac{dy}{dx} = 0[/tex]

[tex]\frac{dy}{dx} = \frac{-16}{30} = \frac{-8}{15}[/tex]

So the answer is choice a)

(93 points) Eloise bought 2 boxes of crackers to share with her friends. Her friends ate 12 of the first box and 34 of the second box.

A. 1/2 box

B. 2/3 box

C. 3/4 box

D. 1 1/3

Answers

Answer:

The answer is C, 3/4 of a box

a cylindrical water tank with its circular base parallel to the ground is being filled at the rate of 4 cubic feet per minute. the radius of the tank is 2 feet. how fast is the level of the water in the tank rising when the tank is half full? give your answer in feet per minute.

Answers

In the given cylinder we know that it takes the water around 3.14 minutes to rise 1 ft.

What is a cylinder?

One of the most fundamental curvilinear geometric shapes, a cylinder has historically been a three-dimensional solid.

It is regarded as a prism with a circle as its base in basic geometry.

In several contemporary fields of geometry and topology, a cylinder can alternatively be characterized as an infinitely curved surface.

So, we know that:

The rate at which water is being filled is 4 ft³.

The radius of the cylinder is 2 ft.

Now, the time in which the level of water in the tank rises when the tank is half full:

Area of cylinder: πr²

πr² =  12.566 ft³

Next, multiply 4 by 12.566 to get how quickly the water is rising:

12.566 ft³/4 ft³/min = 3.14 m

According to this, a foot of water rises every roughly 3.14 minutes, or 1/3.14 - 0.318 ft/min, or 3.82 in/min.

Therefore, in the given cylinder we know that it takes the water around 3.14 minutes to rise 1 ft.


Know more about a cylinder here:

https://brainly.com/question/29953553

#SPJ4

An airplane is descending from 35600 ft at 3200ft per mile,

how long would it be until the plane reaches 18,000 ft? I dont remember the exact ft but i just need a example such as a equation to solve please! please help! T>T

Answers

Answer:

To calculate how long it will take for the plane to reach 18,000 ft, first find the difference between the plane's current altitude and its target altitude:

35,600 ft - 18,000 ft = 17,600 ft

Then, divide this difference by the rate of descent to find the time it will take for the plane to reach its target altitude:

17,600 ft / 3,200 ft/mile = 5.5 miles

Since the rate of descent is given in feet per mile, this result is equivalent to the time it will take for the plane to reach its target altitude. Therefore, it will take the plane 5.5 miles (or a similar amount of time) to reach 18,000 ft.

If k is a negative integer, which of these is DEFINITELY NEGATIVE? A. k* (k-1) * (k - 2) B. k* (k+1) C. k* (-50) D. (50-k)​

Answers

Answer:

only A ans bellow

Step-by-step explanation:

let k= -4

A. k * (k - 1) * ( k - 2)

= -4 * (-4 -1) * ( -4 -2)

= -4* (-5) * (-6)

= 20*-6

= -120

B. k * ( k+1)

= -4 * ( -4+1)

= -4 * (-3)

= + 12

C. k * (-50)

= -4 * (-50)

= + 200

D . (50 - k)

= 50 - (-4)

= 50 + 4

= + 54

Mark me the brainliest

if k is negative then k(k-1)(k - 2) will be definitely negative.

What is Number system?

A number system is defined as a system of writing to express numbers.

Given that k is a  negative integer.

We need to find which of the given options are defnitely negative.

Let us consider k as -3.

k(k-1)(k - 2)

-3(-3-1)(-3-2)

-3(-4)(-5)=-60

Which is negative.

k(k+1)=-3(-3+1)=6 +ve

k (-50)=-3(-50)=150 +ve

(50-k)​=50-(-3)=53 +ve.

Hence, if k is negative then k(k-1)(k - 2) will be definitely negative.

To learn more on Number system click:

https://brainly.com/question/22046046

#SPJ2


Write an equation that represents the line.
Use exact numbers.

Answers

answer : y = 4/5x - 22/5

y = mx + b

get m or slope (aka "rate" or "speed")

(fun fact : see how the graph is a straight line? a lot of calculus is trying to get the rate or speed or slope of a curved graph)

(-3,-6) (2,-2)

(y2 - y1)/(x2-x1)

-2 + 6/2 + 3

m = 4/5

(if you make the 2 points a triangle bottom is 5 and the right side is 4)

line formula

y - y1 = m (x - x1)

y + 6 = 4/5 ( x + 2 )

y = 4/5x + 8/5 - 6

y = 4/5x + 8/5 - 30/5

y = 4/5x - 22/5

side note : since the graph doesnt go thru (0,0) the graph is NOT PROPORTIONAL

pool workers conducted a chi-square goodness-of-fit test at the 5% significance level. the chi-square value is 1.3892 and the p-value is 0.7081. what can the pool workers conclude?

Answers

The pool workers can conclude that there is not a statistically significant difference between the expected and observed results.

The chi-square value is 1.3892 and the p-value is 0.7081 The p-value of 0.7081 is greater than the 5% significance level, meaning that the null hypothesis can not be rejected.

This means that the difference between the expected and observed results is not statistically significant, and the observed results are likely due to random chance. This means that there is not a significant difference in the observed results from the expected results, and the null hypothesis is accepted.

For more questions like Chi-square  click the link below:

https://brainly.com/question/16749509

#SPJ4

How do I solve for x in #34 and #35?

Answers

Answer:

  34. x = 3 1/3

  35. x = 5.2

Step-by-step explanation:

Given figures involving triangles with angle bisectors and parallel segments, you want to solve for x.

In each case, you need to make use of two proportional relationships. An angle bisector divides the triangle proportionally. An segment parallel to one side of the triangle divides it proportionally.

34.

Using the angle bisector relation, you have ...

  QP/PT = QS/ST

  x/3 = 5/ST   ⇒   x = 15/ST

Using the parallel segment relation, you have ...

  PT/QR = ST/SR

  3/2 = ST/3   ⇒   ST = 9/2

Using the value of ST in the equation for x gives ...

  x = 15/(9/2) = 30/9 = 10/3

  x = 3 1/3

35.

Using the angle bisector relation, you have ...

  EF/ED = CF/CD

  7.2/9 = CF/6   ⇒   CF = 6(7.2/9) = 4.8

Using the parallel segment relation, you have ...

  CB/BA = CF/FE

  x/7.8 = 4.8/7.2

  x = 7.8(4.8/7.2)

  x = 5.2

__

Additional comment

In each case, we assigned a numerical value to the intermediate variable (ST, CF). We didn't actually need to do that.

You know distance and speed and want to find the time

You know time and distance and want to find the speed

You know speed and time and want to find the distance

Answers

Answer:

time=speed÷distance

speed=distance×time

distance=speed ÷time

Step-by-step explanation:

from the answers we get a relationship between speed,distance and time.there by when with two you can find the other

Establish the identity.
1+ tan^2 (-0) = sec^2 0
Which of the following four statements establishes the identity?

pls help asap i can’t pass this class without passing this test :(

Answers

The correct statement that establishes the identity is the third one: "The reciprocal of the cosine of an angle is equal to the secant of the angle."

Here's how you can prove this using the other three statements:

"The tangent of an angle is equal to the sine of the angle divided by the cosine of the angle."

"The reciprocal of the sine of an angle is equal to the cosecant of the angle."

"The reciprocal of the cosine of an angle is equal to the secant of the angle."

"The reciprocal of the tangent of an angle is equal to the cotangent of the angle."

To prove the identity, we can start by substituting the first statement into the equation on the left side of the identity:

$1 + \tan^2 (-0) = 1 + \left( \frac{\sin (-0)}{\cos (-0)} \right)^2 = 1 + \frac{\sin^2 (-0)}{\cos^2 (-0)}$

Next, we can use the fourth statement to write the right side of the identity in terms of the cotangent:

$1 + \frac{\sin^2 (-0)}{\cos^2 (-0)} = 1 + \frac{1}{\cot^2 (-0)} = \frac{1}{\cot^2 (-0)}$

Now we can use the second statement to write the right side of the identity in terms of the cosecant:

$\frac{1}{\cot^2 (-0)} = \frac{1}{\frac{1}{\sin^2 (-0)}} = \csc^2 (-0)$

Finally, we can use the third statement to write the right side of the identity in terms of the secant:

$\csc^2 (-0) = \frac{1}{\sec^2 (-0)} = \sec^2 (-0)$

Therefore, the third statement establishes the identity.

Other Questions
all of the following are key functions of operating a business except: select one: a. economics b. management c. accounting d. finance. e. marketing What is 2 to the power of three halves equal to? square root of 8 cube root of 8 cube root of 16 square root of 16 find constant rate of change Select all the values of x that make the equation | 3x + 4 | 9 = 0 true.A . 1 2/3B. -1 2/3C. 4 1/3D. -4 1/3 E. 5 2/3F. -5 2/3 4. If s between a basketball shoe and a court is 0.56, and the normal reactionforce acting on the shoe is 350 N, how much horizontal force is required tocause the shoe to slide? a message that signals to recipients that an upcoming conversation may involve unpleasant news is called a ida wells launched a campaign against what practice that made her one of the most powerful black activists of her era? jessica has just taught maurice how to change the oil in maurices car. Maurice is thrilled to learn something new and says, "thanks for enlightening me!" how does maurices concept of enlightenment likely related to the jain concept if enlightenment? how is it different? explain your answer. At a real estate agency, an agent sold a house for $306,000. The commission rate is 7.5% for the real estate agency and the commission rate for the agent is 20% of the amount the real estate agency gets. How much did the agency make on the house? How much did the agent earn in commission?The agency made (answers) on the house refer to fact pattern 29-1a. nani, a professional driver, buys a customized luxuro from motorpros to drive in a grand prix race. nani's luxuro is according to one model, stress is the result of systemic activation of the hypothalamus, the pituitary gland, and the [ select ] . the result of these series of activations is the release of hormones. the main stress hormone in this release is called [ select ] . which of the following is not a risk factor for exertional heat illness for students?WeightAgeLevel of fitnessPrior recent illnessAmount of hydration before activity How do the photoperiodism and vernalization affect the flowering and dormancy of plants? Heeeeelp I am so confuseeeeddlet's just say there's a planet that has 4 satellitesif our Moon rotates from W to E and revolves from W to E around the earth,then what is the rotating&revolving motions of these 4 moons??moon 1: rotates from W to E and revolves the opposite way (like Triton)moon 2: rotates from W to E and revolves from W to E -- the regular moonmoon 3: rotates the opposite way and revolves from W to Emoon 4: rotates the opposite way and revolves the opposite way -- the exact reversed moonin my memory, moon 3 is making the retrograde motion and moon 2 is making the prograde motion....but suddenly I got so confused about which moon is making the retrograde motion is it moon 1, 3, or 4??? if you figured it out, tell me also about the other moon's motions then I'll pick you as a brainliest 5 . the bank charged a service fee of $55. how would this information be included on the bank reconciliation? a. a deduction on the book side b. an addition on the book side c. an addition on the bank side d. a deduction on the bank side which of the following are effective delivery cues recommended for speakers to signal that their speech is about to end? The operon model of the regulation of gene expression in bacteria was proposed by _____.Watson and CrickFranklinDarwinJacob and MonodMendelJacob and Monod A rocket motor is manufactured by bonding together two types of propellants, an igniter and a sustainer. The shear strength of the bond (y) is thought to be a linear function of the age of the propellant (X) when the motor is cast. Twenty observations are shown in the following table.A) find the least squares estimated of the slope and intercept in the linear regression modelB) find the estimate of (?^2)C) estimate the mean shear strength of a motor made from propellant that is 20 weeks old which cells function as professional antigen presenting cells? multiple answers apply and all should be chosen. a volleyball is spiked so that its incoming velocity of 3.74 m/s is changed to an outgoing velocity of -19.7 m/s. the mass of the volleyball is 0.350 kg. what is the magnitude of the impulse that the player applies to the ball?